Finding a matrix to represent a 2x2 transpose mapping

Click For Summary
The discussion focuses on finding a matrix representation for the transpose mapping L(A) = A^t with respect to the standard basis. Participants clarify that the standard basis should consist of unit vectors rather than the vector [1, 1, 1, 1]. A 4x4 matrix is deemed appropriate for this mapping, and one participant proposes a specific matrix configuration. The solution is confirmed as valid, provided that the context allows for working with vectors in R^4 instead of 2x2 matrices. Overall, the conversation emphasizes the importance of correctly identifying the basis and the dimensionality of the matrices involved.
PsychonautQQ
Messages
781
Reaction score
10

Homework Statement


Let L be a mapping such that L(A) = A^t, the transpose mapping. Find a matrix representing L with respect to the standard basis [1,1,1,1]

Homework Equations

The Attempt at a Solution


So should I end up getting a 4x4 matrix here? I got 1,0,0,0 for the first column, 0,0,1,0 for the second column, 0,1,0,0 for the third column and 0,0,0,1 for the fourth column. is this correct?
 
Physics news on Phys.org
PsychonautQQ said:

Homework Statement


Let L be a mapping such that L(A) = A^t, the transpose mapping. Find a matrix representing L with respect to the standard basis [1,1,1,1]
How is [1, 1, 1, 1] a basis?
I think you mean {<1, 0, 0, 0>, <0, 1, 0, 0>, <0, 0, 1, 0>, <0, 0, 0, 1>}.
PsychonautQQ said:

Homework Equations

The Attempt at a Solution


So should I end up getting a 4x4 matrix here? I got 1,0,0,0 for the first column, 0,0,1,0 for the second column, 0,1,0,0 for the third column and 0,0,0,1 for the fourth column. is this correct?
This works, if it's legitimate to work with vectors in ##\mathbb{R}^4## instead of 2 x 2 matrices. Of course ##\mathbb{M}_{2 x 2}## is isomorphic to ##\mathbb{R}^4##. Based on what I think the problem statement is supposed to mean, your solution looks fine.
 
Question: A clock's minute hand has length 4 and its hour hand has length 3. What is the distance between the tips at the moment when it is increasing most rapidly?(Putnam Exam Question) Answer: Making assumption that both the hands moves at constant angular velocities, the answer is ## \sqrt{7} .## But don't you think this assumption is somewhat doubtful and wrong?

Similar threads

Replies
5
Views
2K
  • · Replies 3 ·
Replies
3
Views
3K
  • · Replies 4 ·
Replies
4
Views
1K
  • · Replies 4 ·
Replies
4
Views
2K
Replies
9
Views
3K
Replies
3
Views
3K
  • · Replies 1 ·
Replies
1
Views
2K
  • · Replies 2 ·
Replies
2
Views
4K
Replies
3
Views
2K
  • · Replies 5 ·
Replies
5
Views
2K